The Leading Independent
Resource for Top-tier MBA
Candidates
Home » Blog » GMAT » GMAT Tips » GMAT - Verbal » GMAT Tips: Breaking Down a GMATPrep Critical Reasoning Strengthen Problem

GMAT Tips: Breaking Down a GMATPrep Critical Reasoning Strengthen Problem

Today’s GMAT tip comes from test prep firm ManhattanGMAT. In this article, they provide helpful tips for how to identify and answer “strengthen” questions in the Critical Reasoning section of the GMAT.  Read on to see what they have to say!

This week, we’re going to tackle a harder GMATPrep critical reasoning question from the Strengthen category.

Let’s start with the problem. Set your timer for 2 minutes!

* “Sales of telephones have increased dramatically over the last year. In order to take advantage of this increase, Mammoth Industries plans to expand production of its own model of telephone, while continuing its already very extensive advertising of this product.

“Which of the following, if true, provides most support for the view that Mammoth Industries cannot increase its sales of telephones by adopting the plan outlined above?

“(A) Although it sells all of the telephones that it produces, Mammoth Industries’ share of all telephone sales has declined over the last year.

“(B) Mammoth Industries’ average inventory of telephones awaiting shipment to retailers has declined slightly over the last year.

“(C) Advertising has made the brand name of Mammoth Industries’ telephones widely known, but few customers know that Mammoth Industries owns this brand.

“(D) Mammoth Industries’ telephone is one of three brands of telephone that have together accounted for the bulk of the last year’s increase in sales.

“(E) Despite a slight decline in the retail price, sales of Mammoth Industries’ telephones have fallen in the last year.”

Okay, now that you’ve got an answer, we’re going to go back to the question stem and argument and forget about the answers for a moment. How do you identify the question type? What should you know about that question type before you even start to read the argument? What should you look for in the argument? What might your notes have looked like?

Reading the Question and Argument

First, if you haven’t already, you may want to take a look at this article: Strategies for Critical Reasoning.

As the article says, we read the question stem first. This is the question stem:

“Which of the following, if true, provides most support for the view that Mammoth Industries cannot increase its sales of telephones by adopting the plan outlined above?”

The key identifying language is pretty straightforward on this one: “provides most support for <the conclusion>.” (Note: if I had written the problem myself, I would have said “provides the most support. I’m guessing the missing “the” is a typo, but I’m quoting right from the software, and that’s what it says.)  This language reflects the Strengthen problem type.

What kind of information is found in Strengthen questions and what are we supposed to do with that information?

Strengthen questions ask us to find the answer choice that makes the conclusion at least a little bit more likely to be true. We do not have to find something that makes the conclusion definitely true, or even really strong. The correct answer will also represent a new (but closely related) piece of information. This is unlike assumption or conclusion / inference questions, where new information typically means the answer choice is out of scope.

In this argument, they gave us a critical piece of information in the question stem: someone holds the view that, if Mammoth Industries adopts a certain plan, it will not sell more telephones. Two things are important here: first, they’re giving us the conclusion in the question stem, not in the argument. Second, the conclusion is NOT what Mammoth plans to do or the idea that the plan will succeed. Somebody else is concluding that Mammoth’s plan won’t succeed and we’re trying to strengthen that negative idea.

Notice that underlined word “cannot” in the question stem. When they give negatively worded questions, they will underline the negative word or write it in capital letters. You may see the word “except,” “not,” or even “cannot,” as we do in this problem. Make sure you note that on your scrap paper!

Okay, now let’s take a look at the argument itself. The first sentence gives a fact: telephone sales went up a lot last year. The second sentence gives us Mammoth’s plan to “take advantage of” this sales increase: the company will make more phones and continue the same (extensive) advertising that it has already been doing.

Your notes might look something like this (though there are lots of ways to write notes!):

LY sales incr lot (or draw two arrows pointing up for “incr lot”)

M make > phones, keep lots of ads

BUT it won’t work – why?

Clearly, Mammoth thinks it’s going to get some advantage out of making more phones; presumably, the company thinks they’ll actually be able to sell more. But the author thinks that they’re not actually going to be able to sell any extra phones, and we have to find something that supports (at least a little bit) the author’s point of view.

If any assumptions jump to mind right now, add them to your notes. For example, Mammoth is assuming that, if it makes more phones available, people will choose to buy Mammoth’s phones over some other phones. Will they? Maybe Mammoth has a bad reputation and people don’t want their phones.

Okay, now we know what we need to do. There’s just one more thing: the trap wrong answers for this question type.

On Strengthen questions, the most tempting traps typically address some part of the premises but don’t actually affect the conclusion. Further, on negatively-worded questions, we’ll almost always have at least one trap answer that involves doing the opposite of what the question asked (in this case, strengthening Mammoth’s plan, instead of saying Mammoth won’t succeed).

Answering the Question

To recap: we first want to find the argument’s conclusion. Then, we want to find the answer choice that makes that conclusion a little bit more likely to be true.

In this problem, the conclusion is that Mammoth’s plan won’t work; the company won’t be able to sell more phones. We also brainstormed an assumption that had to do with whether people wanted to buy Mammoth’s phones – if people don’t want to buy the phones in the first place, then making more phones doesn’t really help.

Now, go through the answers and eliminate any that you think are definitely wrong. For any that might be right, don’t eliminate yet; save them for a later comparison.

Answer A says that Mammoth’s share of telephone sales declined over the last year. That sounds promising – maybe people don’t like Mammoth’s phones, as we thought when we were brainstorming. Leave this one in.

Answer B talks about inventory… hmm. What does that have to do with sales? Well, it might have something to do with sales. Or maybe they just changed their inventory or shipping procedures. I’d have to make a bunch of additional assumptions to tie inventory to sales. No good – eliminate B.

Answer C talks about branding. Having a “widely known” brand is generally a good thing (unless the brand has a negative reputation, and they don’t tell me that here). Plus, who cares whether people know that Mammoth owns the brand? Eliminate C.

Answer D says that Mammoth’s phone was one of three brands that experienced an increase in sales last year. That’s good for Mammoth. In fact, if Mammoth was already able to sell more phones last year, when people were buying more, the plan to make more phones this year makes total sense. Ah, this is an “opposite” trap – it’s strengthening Mammoth’s plan. Eliminate D.

Answer E says that Mammoth’s sales fell last year; that’s not good. There was an overall market increase in the number of phones sold last year, and yet Mammoth sold fewer phones. That’s sounding pretty good for our author’s conclusion, too. Keep this one in.

B, C, and D are out. We need to examine A and E again. For both of those answers, we really only examined the main part of each sentence; each one contained some additional info, so let’s dig in a little deeper.

Answer A also tells us that Mammoth currently sells all of the phones that it produces; in other words, even though the company’s share of the market fell last year, it wasn’t because people didn’t want to buy their phones. It isn’t even the case that Mammoth sold fewer phones than usual last year; it might even have sold more, but other companies might also have sold more and thereby taken a greater percentage of the overall market. Actually, that first part of answer A helps Mammoth’s plan – if it is able to sell all of the phones it makes, then there’s reason to believe that it could sell even more if it made more.

Answer E begins by telling us that the price for a Mammoth phone has decreased in the last year. That’s not a good sign. On top of that, the company sold fewer phones. The price was reduced, and yet they still sold fewer phones, even though more phones overall were sold in the market? That sounds pretty bad for Mammoth –people already don’t want to buy the phones at a discount! How will making more phones help?

It probably won’t, which is the author’s point. The correct answer is E.

Key Takeaways for Solving Strengthen CR Problems:

(1) Know how to recognize this type. This is typically straightforward, as most of these will actually use some form of the word “support” or “strengthen” in the question stem. The one tricky spot: sometimes, draw a conclusion questions will also use the word support (for example, “the argument above best supports which of the following statements?”). A Strengthen question is asking us to find support in the answer choices for a conclusion in the argument. A Draw a Conclusion question does the reverse: we use the argument above as support for a conclusion found in the answers.

(2) Know what to do with Strengthen questions. Find the conclusion and identify the main supporting premise. If you can articulate any gaps (assumptions) between the main premise and the conclusion, do so. Our job is to find a new piece of information that makes the conclusion at least a little bit more likely to be true.

(3) Watch out for traps! The correct answer does have to affect the conclusion, not just a premise. Also, some incorrect answers will do the opposite of what we want (in this case, weaken rather than strengthen). Be careful!

* GMATPrep questions courtesy of the Graduate Management Admissions Council. Usage of this question does not imply endorsement by GMAC.

For more information on ManhattanGMAT, download Clear Admit’s independent guide to the leading test preparation companies here. This FREE guide includes coupons for discounts on test prep services at ten different firms!

Clear Admit
The leading resource for top-tier MBA candidates.